You are on page 1of 56

PHT

1. CS Select the range of the normal pressure in the pulmonary artery:


a) [ ] 45-60 mm Hg
b) [x] 25 - 30 mm Hg
c) [ ] 60 - 100 mm Hg
d) [ ] 100 - 120 mm Hg
e) [ ] 60 - 120 mmHg
---------------------------------------------------------------------
2. CS CREST syndrome includes the following, except:

a) [ ] subcutaneous calcinosis
b) [ ] esophageal dysfunction
c) [ ] diaphragm dysfunction
d) [ ] sclerodactily, telangiectasies
e) [x] clubbing
---------------------------------------------------------------------
3. CS Pathogenetic mechanisms of PAH in scleroderma are the following, except:

a) [ ] increased pulmonary flow


b) [ ] vasoconstriction
c) [ ] pulmonary vascular wall remodeling
d) [ ] thrombosis in situ
e) [x] deficiency in synthesis of vasoconstrictive substances
---------------------------------------------------------------------
4. CS from the following the mechanism of pulmonary hypertension in splenectomised patients:

a) [x] increased life of red blood cells and platelet activity


b) [ ] inhibition of K+ channel
c) [ ] increased cardiac output
d) [ ] decreased cardiac output
e) [ ] portal hypertension
---------------------------------------------------------------------
5. 7. CS According to the functional classification, second class of PAH corresponds to the following
statement:

a) [x] slight limitation of exercise capacity. No symptoms at rest and ordinary physical activity causes symptoms
(dyspnea, chest pain, presyncope, fatigue)
b) [ ] there is no limitation of exercise capacity. Ordinary physical activity does not cause symptoms
c) [ ] marked limitation of exercise capacity. No symptoms at rest and less than ordinary physical activity causes
symptoms
d) [ ] inability to perform physical activity. They may have symptoms at rest, increased by minimal effort
e) [ ] none of the above
---------------------------------------------------------------------
6. CS Hemodynamic classification of PAH includes the following, except:

a) [x] decreased pulmonary blood flow


b) [ ] precapillary compartment (arteries)
c) [ ] postcapillary compartment (veins)
d) [ ] mixed
e) [ ] Increased pulmonary flow
---------------------------------------------------------------------
7. CS Select the range of moderate PAH:

a) [ ] 25-40 mm Hg
b) [x] 41-75 mm Hg
c) [ ] 76-110 mm Hg
d) [ ] > 110 mm Hg
e) [ ] None of the above
---------------------------------------------------------------------
8. CS Mild PAH has the following values:

a) [x] 25-40 mm Hg
b) [ ] 41-75 mm Hg
c) [ ] 76-110 mm Hg
d) [ ] > 110 mm Hg
e) [ ] > 130 mm Hg
---------------------------------------------------------------------
9. CS Severe PAH has the following values:

a) [ ] 25-40 mm Hg
b) [ ] 41-75 mm Hg
c) [x] 76-110 mm Hg
d) [ ] > 110 mm Hg
e) [ ] > 15 mm Hg
---------------------------------------------------------------------
10. CS Very severe PAH has the following values:

a) [ ] 25-40 >110 mm Hg
b) [ ] 41-75 >110 mm Hg
c) [ ] 76-110 >110 mm Hg
d) [x] >110 mm Hg
e) [ ] > 15
---------------------------------------------------------------------
11. CS ECG signs in PAH are the following, except:

a) [ ] right ventricular hypertrophy


b) [ ] right atrial hypertrophy
c) [ ] incomplete right bundle branch block
d) [ ] P- pulmonale
e) [x] elongation PQ
---------------------------------------------------------------------
12. CS Select the method of choice that helps to exclude pulmonary thromboembolism:

a) [ ] chest radiograph
b) [ ] EchoCG
c) [ ] Electrocardiography
d) [x] pulmonary scintigraphy
e) [ ] ultrasound examination
---------------------------------------------------------------------
13. CS Pulmonary angiography in PAH is indicated in patients with:

a) [ ] pulmonary edema
b) [x] suspected PAH in chronic thromboembolism
c) [ ] interstitial pulmonary fibrosis
d) [ ] bronchiectasis
e) [ ] arterio-venous shunt
---------------------------------------------------------------------
14. CS The SaO2 value indicating necessity in oxygen therapy is:

a) [ ] less than 99%


b) [ ] less than 95%
c) [x] less than 90%
d) [ ] <70%
e) [ ] <60%
---------------------------------------------------------------------
15. CS Clinical signs of right ventricular dysfunction are the following, except:

a) [x] pulmonary crackles


b) [ ] distended neck veins
c) [ ] positive venous pulse
d) [ ] hepatomegaly
e) [ ] peripheral edema
---------------------------------------------------------------------
16. CS Causes of cor pulmonale are:

a) [ ] COPD
b) [ ] Idiopathic pulmonary fibrosis
c) [ ] Bronchial Asthma
d) [ ] idiopathic pulmonary hypertension
e) [x] all of the listed
---------------------------------------------------------------------
17. CS Pickwick syndrome include the following, except:

a) [ ] drowsiness
b) [ ] obesity
c) [ ] plethora
d) [ ] edema
e) [x] situs viscerus inversus
---------------------------------------------------------------------
18. CS Sleep apnea includes the following manifestations, except:

a) [ ] daytime sleepiness
b) [ ] repeated episodes of apnea during night
c) [ ] improve on CPAP treatment
d) [ ] morning headaches
e) [x] cough
--------------------------------------------------------------------
19. CS Symptoms of cor pulmonale are, except:

a) [ ] cough (with or without sputum), wheezing


b) [ ] central cyanosis
c) [ ] syncope on exertion
d) [ ] angina-like chest pain
e) [x] paresthesia
---------------------------------------------------------------------
20. CM Lung diseases cause PAH through the following mechanisms:

a) [ ] ischemia
b) [x] hypoxia
c) [x] inflammation
d) [ ] bronchial obstruction
e) [x] loss of pulmonary vessels
---------------------------------------------------------------------
21. CM What are the mechanisms by which pulmonary hypertension induce portal hypertension:

a) [x] increased lung flow secondary to increased cardiac output


b) [x] increasing the friction forces
c) [x] vascular remodeling
d) [ ] decreased cardiac output
e) [ ] hypertension
---------------------------------------------------------------------
22. CM Classic symptoms of PAH are:

a) [x] dyspnea
b) [x] angina-like chest pain
c) [x] syncope with exercise
d) [ ] hemoptysis
e) [ ] fatigue
---------------------------------------------------------------------
23. CM Right ventricular failure is reflected by:

a) [x] increased central venous pressure


b) [ ] increased central arterial pressure
c) [x] ascites
d) [x] peripheral edema
e) [ ] pulmonary edema
---------------------------------------------------------------------
24. CM The differential diagnosis of PAH includes the following:

a) [ ] systemic hypertension
b) [x] left heart failure
c) [x] left atrial myxoma
d) [x] pulmonary artery stenosis
e) [ ] mitral
---------------------------------------------------------------------
25. CM The conventional treatment of PAH include:

a) [x] oxygen
b) [x] cardiac glycoside agents
c) [x] diuretic agents
d) [x] anticoagulation
e) [ ] nitrates
---------------------------------------------------------------------
26. CM Non-conventional treatment in PAH group 1 (idiopathic PAH, familial due to diseases affecting the
muscular pulmonary arteries includes administration of:

a) [x] calcium antagonists


b) [x] prostacyclines
c) [x] endothelin receptor antagonists
d) [x] phosphodiesterase inhibitors
e) [ ] nitrate
---------------------------------------------------------------------
27. CM Treatment for thromboembolic pulmonary hypertension includes:

a) [x] long term anticoagulation


b) [x] trombarterectomy
c) [ ] oxygen therapy
d) [ ] angiotensin-converting enzyme inhibitors
e) [ ] beta blockers
---------------------------------------------------------------------
28. CM Diagnostic criteria for cor pulmonale are:

a) [x] PAH proven by any method


b) [x] proven right ventricular hypertrophy
c) [x] dilated right ventricle confirmed by imaging methods
d) [x] clinical signs of right ventricular dysfunction
e) [ ] systolic pulmonary outbreaks
---------------------------------------------------------------------
29. CM Obstructive pulmonary diseases causing cor pulmonale are:

a) [x] chronic bronchitis


b) [x] pulmonary emphysema
c) [x] asthma
d) [x] cystic fibrosis
e) [ ] idiopathic pulmonary fibrosis

30. CM Alveolar hypoventilation causes:

a) [x] hypercapnia
b) [x] hypoxemia
c) [ ] hypocapnia
d) [ ] respiratory alkalosis
e) [ ] none of the mentioned
--------------------------------------------------------------------
31. CM Impaired respiratory center causes:

a) [x] hypoventilation
b) [x] respiratory acidosis
c) [ ] hypocapnia
d) [x] hypoxemia
e) [x] pulmonary hypertension
---------------------------------------------------------------------
32. CM Complications of cor pulmonale are:
a) [x] pulmonary thromboembolism
b) [ ] myocardial infarction
c) [x] arrhythmias
d) [x] relative tricuspid insufficiency
e) [ ] tricuspid stenosis
---------------------------------------------------------------------
33. CM Treatment options for cor pulmonale are:

a) [x] diuretics
b) [x] digitalics
c) [ ] antileukotriens
d) [ ] corticosteroids
e) [x] phlebotomies
---------------------------------------------------------------------
34. CM Methods for evaluation of pulmonary hypertension are:

a) [x] measurement of central venous pressure


b) [ ] ECG
c) [x] pulmonary artery catheterization
d) [x] measurement of right ventricular pressure
e) [x] EchoCG
---------------------------------------------------------------------

ASTHMA
35. CS Clinical differentiation between COPD and asthma is based on:
a) [ ] a long history of coughing
b) [ ] prolonged wheezing past
c) [x] complete reversibility of bronchial obstruction after bronchodilator therapy
d) [ ] onset of cough with expectoration
e) [ ] the presence of mucopurulent sputum
--------------------------------------------------------------------
36. CS In a patient with asthma, marked chest hyperinflation, involvement of accessory muscles and the
presence of a pulsus paradoxus suggest:
a) [x] severe airway obstruction
b) [ ] usual asthma attack
c) [ ] pneumonia
d) [ ] pulmonary embolism
e) [ ] an overimposed infection
---------------------------------------------------------------------
37. CS Drugs most commonly associated with the induction of asthma attacks are:

a) [ ] ACE inhibitors
b) [x] NSAIDs
c) [ ] calcium blockers
d) [ ] beta-lactam antibiotics
e) [ ] corticosteroids
---------------------------------------------------------------------
38. CS Select the typical clinical finding on examination of a patient during asthma attack:

a) [x] wheezing
b) [ ] sweating
c) [ ] nasal and conjunctival hypersecretion
d) [ ] distended neck veins
e) [ ] bradyarrhythmias
---------------------------------------------------------------------
39. CS Select the drug suitable for treatment in asthma:

a) [ ] cholinomimetics
b) [ ] antibiotics
c) [x] beta agonists
d) [ ] beta blockers
e) [ ] alpha blockers
---------------------------------------------------------------------
40. CS The diagnosis of asthma is based on:

a) [x] reversible airway obstruction


b) [ ] positive skin tests to different allergens
c) [ ] blood eosinophilia
d) [ ] sputum eosinophilia
e) [ ] increased IgE in serum
---------------------------------------------------------------------
41. CS Sodium cromoglycate is used in the treatment of asthma because of its ability to:

a) [x] stabilize the mast cell membranes


b) [ ] cause bronchodilatation
c) [ ] induce anti-infective protection
d) [ ] facilitate expectoration
e) [ ] loosen mucus
---------------------------------------------------------------------
42. CS Treatment of choice for intermittent asthma include:
a) [x] inhaled beta-agonists to relief symptoms
b) [ ] long-acting inhaled beta-agonists
c) [ ] long term inhaled corticosteroids
d) [ ] long term administration of beta-agonists, corticosteroids and mast cell stabilizers
e) [ ] long term administration of oral corticosteroids
---------------------------------------------------------------------
43. CS Airway reversibility is conventionally defined as an increase of FEV1 after inhaling a beta agonist with:

a) [ ] <5%
b) [ ] 5% - 10%
c) [ ] <10%
d) [ ] 10% - 15%
e) [x] > 15% and 200ml
---------------------------------------------------------------------
44. CS The most effective treatment for asthma attacks includes:

a) [ ] mast cell stabilizers


b) [ ] inhaled corticosteroids
c) [ ] anti-cholinergic medication
d) [ ] oral corticosteroids
e) [x] inhaled beta-2-agonists
---------------------------------------------------------------------
45. CS Anti-cholinergics used in the treatment of asthma have the following disadvantages:

a) [x] slow action - in 60-90 minutes


b) [ ] cause tachycardia
c) [ ] only parenteral use
d) [ ] liver toxicity
e) [ ] contraindicated in preexisting renal disease
---------------------------------------------------------------------
46. CS Select the group of drugs that should be avoided in patients during an asthma attack:

a) [ ] diuretics
b) [x] sedatives
c) [ ] beta-mimetics
d) [ ] antibiotics
e) [ ] antipyretics
---------------------------------------------------------------------
47. CS Asthma is defined as a disease that primarily affects:

a) [ ] pulmonary alveoli
b) [ ] the larynx
c) [x] the airways
d) [ ] the lung interstitium
e) [ ] the airways and lung interstitium
---------------------------------------------------------------------
48. CS Name the changes in pulmonary function tests consistent with asthma, except:

a) [ ] decrease in FEV1
b) [ ] normal or decreased FVC
c) [ ] increased RV
d) [x] decreased TLC
e) [ ] increased FRC
---------------------------------------------------------------------
49. CS Bronchial reactivity in asthma, is triggered by:
a) [ ] allergens
b) [ ] viral infections
c) [ ] bacterial infections
d) [ ] exercise
e) [x] a variety of stimuli
---------------------------------------------------------------------
50. CS Name the correct answer about the obstruction in status asthmaticus:

a) [ ] is mild
b) [ ] is resolves spontaneously
c) [ ] lasts for minutes
d) [x] can be lethal
e) [ ] is can be managed with usual therapy
---------------------------------------------------------------------
51. CS Name the correct answer about the epidemiology of asthma:
a) [ ] it is a rare disease
b) [x] the condition is very common
c) [ ] it is mainly encountered in women
d) [ ] it affects only children
e) [ ] it affects only elderly persons
---------------------------------------------------------------------
52. CS The diagnosis of asthma is supported by:

a) [x] reversibility of airway obstruction


b) [ ] eosinophilia in sputum and blood
c) [ ] increased serum IgE
d) [ ] hyperinflation on a chest Xray
e) [ ] positive skin tests to various allergens
--------------------------------------------------------------------
53. CS Name the criteria for assessment of the level of control in asthma patients, except:

a) [ ] daytime symptoms
b) [ ] night symptoms
c) [ ] frequency of “relievers” drugs use
d) [ ] number of exacerbations
e) [x] presence of night sweats
---------------------------------------------------------------------
54. CS Name the bronchodilatation mechanism of Ipratropium bromide in asthma:

a) [ ] beta receptor stimulation


b) [ ] reduction of inflammation in the bronchial mucosa
c) [ ] inhibition of phosphodiesterase
d) [ ] inhibition of mast cell degranulation
e) [x] anticholinergic effect
--------------------------------------------------------------------
CS The following drugs are recommended treatment in asthma, except:

a) [x] methotrexate
b) [ ] beta - 2 - agonists
c) [ ] inhaled corticosteroids
d) [ ] mast cell stabilizers
e) [ ] oral corticosteroids
---------------------------------------------------------------------

55. CS The following stimuli trigger airway reactivity and acute episodes of asthma, except:
a) [ ] allergens
b) [x] beta agonists
c) [ ] respiratory infections
d) [ ] smoking
e) [ ] exercise
---------------------------------------------------------------------
56. CS The reversibility of airway obstruction in asthma is defined as:

a) [x] an increase of 15% or more and at least 200 ml in FEV1 after inhaling a beta-agonist
b) [ ] absence of dyspnea after treatment
c) [ ] absence of cough after treatment
d) [ ] an increase of 5% or less of FEV1 after inhaling a beta-agonist
e) [ ] increase of 10% or less of FEV1 after inhaling a beta-agonist
---------------------------------------------------------------------
57. CS Asthmatic triad or Samter’s triad is caused by one of the following drugs:

a) [ ] penicillin
b) [ ] sulfanilamide
c) [ ] zafirlukast
d) [x] aspirin
e) [ ] ondansetron
---------------------------------------------------------------------
58. CS In the pathogenesis of asthma the released active mediators are the following, except:

a) [ ] histamine
b) [ ] bradykinin
c) [ ] serotonin
d) [ ] acetylcholine
e) [x] streptolysin
---------------------------------------------------------------------
59. CS Asthma attacks develop after blocking of following adrenergic receptors:

a) [ ] alpha, beta1, beta2


b) [x] beta-1 and beta-2
c) [ ] beta-1 and alpha
d) [ ] beta and alpha-2
e) [ ] alpha
---------------------------------------------------------------------
60. CS A 15 year old patient came to the ER with attacks of breathlessness that started at home. In childhood,
she was allergic to eggs and cheese manifested by eczema. At the age of 13, these signs have disappeared.
On her 15th birthday, she received a cat as a birthday present. A few months later, she started to experience
mild attacks of dyspnea. What could be a possible diagnosis?

a) [ ] chronic obstructive bronchitis


b) [ ] lobar pneumonia
c) [ ] chronic obstructive pulmonary disease
d) [x] allergic asthma
e) [ ] intrinsic asthma
--------------------------------------------------------------------
61. The preferred route of administration of beta-agonists in asthma is:
a) [x] inhaled
b) [ ] intravenous
c) [ ] intramuscular
d) [ ] subcutaneous
e) [ ] oral
-------------------------------------------------------------------
62. CS Cromoglycate is used in the treatment of asthma because it is a:

a) [x] mast cell membrane stabilizer


b) [ ] bronchodilator
c) [ ] anti-infective agent
d) [ ] expectorant
e) [ ] mucolytic
---------------------------------------------------------------------
63. CS In intermittent asthma, the treatment of choice is represented by:
a) [x] administration of inhaled beta-agonists when needed
b) [ ] long term administration of inhaled beta-agonists
c) [ ] long term administration of inhaled corticosteroids
d) [ ] long term administration of beta-agonists, corticosteroids and mast cell stabilizers
e) [ ] long term administration of oral corticosteroids
---------------------------------------------------------------------
64. CM In asthma, the following mediators are involved in the inflammatory reaction:

a) [x] leukotriens
b) [ ] platelet activating factor
c) [ ] phospholipase A2
d) [x] serotonin
e) [x] bradykinin
--------------------------------------------------------------------
65. CM The following drugs may be associated with the induction of asthma attacks:

a) [x] indomethacine
b) [x] naproxen
c) [ ] captopril
d) [ ] atenolol
e) [ ] amlodipine
---------------------------------------------------------------------
66. CM Pathological findings in asthma can reveal:
a) [x] bronchial smooth muscle hypertrophy
b) [ ] vessels atrophy of the bronchial mucosa and submucosa
c) [x] edema of the bronchial mucosa
d) [x] lymphocytes infiltration of in the bronchial wall
e) [ ] marked thickening of the basement membrane of the bronchial mucosa
---------------------------------------------------------------------
67. CS Choose the correct statements about asthma:

a) [x] asthma is a respiratory disorder characterized by increased bronchial reactivity triggered by a variety of
stimuli
b) [x] asthma is characterized, in terms of pathophysiology, by widespread narrowing of the airways, which may
resolve spontaneously or with treatment
c) [x] asthma is characterized clinically by attacks of dyspnea, cough and wheezing
d) [x] asthma is an episodic disease with acute exacerbations alternating with asymptomatic periods
e) [ ] asthma is a condition that evolves with chronic productive cough
---------------------------------------------------------------------
68. CM The most effective methods of treatment in asthma are the following:

a) [x] allergen avoidance in allergic asthma


b) [x] administration of bronchodilators
c) [x] administration of inhaled corticosteroids
d) [ ] antibiotics
e) [ ] expectorants
--------------------------------------------------------------------
69. CM Inhaled corticosteroids are indicated in the treatment of asthma in the following situations:

a) [x] patients with poor control only with short acting inhaled bronchodilators
b) [ ] mild asthma attack
c) [ ] severe asthma attack
d) [x] asthma with frequent attacks
e) [x] mild persistent asthma associated with allergic rhinitis
---------------------------------------------------------------------
70. CM Allergic asthma is associated with personal or family history of:

a) [x] allergic diseases (rhinitis, urticaria, eczema)


b) [x] positive intradermal skin test to antigens
c) [x] increased levels of IgE in serum
d) [x] positive challenge tests to inhaled antigens
e) [ ] low levels of IgE in serum
---------------------------------------------------------------------
71. CM Patients with nonallergic, intrinsic asthma have the following features:

a) [x] no family or personal history of allergic diseases


b) [x] negative allergic skin tests
c) [x] normal levels of IgE in the serum
d) [ ] bradycardia
e) [ ] changes of BP
---------------------------------------------------------------------
72. CM Differential diagnosis of asthma:

a) [x] carcinoid tumors


b) [x] recurrent pulmonary embolism
c) [x] chronic bronchitis
d) [ ] deep vein thrombosis
e) [ ] paroxysmal tachycardia
---------------------------------------------------------------------
73. CM Corticosteroids are used in the treatment of asthma because of their ability to:

a) [ ] induce bronchodilatation
b) [x] reduce airway inflammation
c) [x] reduce the inflammatory cell infiltration in the bronchial wall
d) [ ] induce bronchoconstriction
e) [ ] inhibit the cough reflex
---------------------------------------------------------------------
74. CM Allergic asthma is often associated with:
a) [x] urticaria
b) [ ] scleroderma like skin reaction
c) [ ] high levels of IgA in the serum
d) [x] atopic dermatitis
e) [x] family history of allergic diseases
---------------------------------------------------------------------
75. CM Intrinsic asthma is characterized by:

a) [ ] positive family history of allergy


b) [x] absence of personal history of allergy
c) [x] negative skin tests
d) [ ] increased IgE levels
e) [x] normal IgE levels
---------------------------------------------------------------------

76. CM In asthma patients, the control treatment includes:

a) [x] long-acting beta2-agonists


b) [x] inhaled corticosteroids
c) [ ] antihistamines
d) [ ] oral corticosteroids
e) [ ] metotrexate
---------------------------------------------------------------------
77. CM During an asthma attack the following drugs should be avoided:

a) [x] opiates
b) [ ] ipratropium bromide
c) [x] sedatives
d) [x] tranquilizers
e) [ ] albuterol
---------------------------------------------------------------------
78. CM Select the correct statements about control treatment in asthma patients:
a) [x] is administered only in patients with persistent asthma
b) [x] is achieved by educating the patient
c) [ ] is achieved by administration of anti-inflammatory drugs only when needed
d) [ ] is achieved by administration of daily oral steroids for patients with residual symptoms or with unstable lung
function
e) [ ] is achieved by avoiding aspirin and NSAIDs
---------------------------------------------------------------------
79. CM Symptoms similar to asthma may be encountered in:

a) [x] acute left ventricular failure


b) [x] endobronchial disease
c) [x] recurrent pulmonary embolism
d) [ ] eosinophilic pneumonia
e) [ ] gallstones
--------------------------------------------------------------------
80. CM Clinically, an asthma attack is manifested by:

a) [x] episodes of breathlessness


b) [x] episodes of coughing
c) [x] episodes of wheezing
d) [ ] episodes of fever
e) [ ] episodes of stridor
---------------------------------------------------------------------
81. CM Asthma is a disease:

a) [x] manifested by episodes of symptoms caused by bronchoconstriction


b) [x] with symptom-free periods
c) [ ] without exacerbations
d) [ ] with minimal damage to the airways
e) [ ] with impaired larynx
-------------------------------------------------------------------
82. CM After an attack, a patient with asthma:

a) [ ] has a fever
b) [ ] has chills
c) [x] can be symptom - free
d) [x] may have a period of several days with a certain degree of obstruction
e) [ ] may present a restrictive syndrome reversible after therapy
--------------------------------------------------------------------
83. CM Status asthmaticus represents:

a) [x] acute severe asthma episode unresponsive to repeated courses of beta-agonists therapy
b) [ ] successive episodes of chills
c) [ ] severe obstruction which persists for weeks
d) [ ] episodes of fever preceded by chills
e) [x] a medical emergency
---------------------------------------------------------------------
84. CM Narrowing the airways is the basic pathophysiological feature of asthma caused by:

a) [x] smooth muscle contraction


b) [ ] vascular congestion
c) [x] bronchial wall edema
d) [x] viscous mucus secretion
e) [ ] smooth muscle relaxation
---------------------------------------------------------------------
85. CM Sputum smears of patients with asthma microscopically may reveal:
a) [x] Charcot - Leyden crystals
b) [x] eosinophils
c) [x] Curschmann spirals
d) [ ] lymphocytes
e) [ ] Lupus rosettes
---------------------------------------------------------------------
86. CM Symptoms consistent with asthma are:

a) [x] expiratory dyspnea


b) [x] wheezing
c) [x] cough with thick, pearl-like sputum
d) [ ] cough with mucopurulent expectoration
e) [ ] chest pain
---------------------------------------------------------------------
87. CM Microscopic examination of sputum in patients with asthma has the following features:

a) [ ] frequent polymorphonuclear cells


b) [x] frequent eosinophils
c) [x] Charcot - Leyden Crystals
d) [ ] common Gram positive microbial flora
e) [ ] common Gram negative microbial flora
---------------------------------------------------------------------
88. CM What possible complications can occur in patients with asthma?

a) [ ] Lung abscess
b) [ ] Interstitial pulmonary fibrosis
c) [x] Cor pulmonale
d) [x] pulmonary emphysema
e) [ ] pulmonary embolism
---------------------------------------------------------------------
89. CM Select the triggers in allergic asthma:

a) [x] household dust


b) [x] animal and human dander
c) [x] bird feathers
d) [ ] humidity
e) [x] plant pollens
---------------------------------------------------------------------
90. CM Predisposing factors for asthma are:

a) [ ] humidity
b) [x] heredity
c) [ ] atmospheric temperature fluctuations
d) [ ] oscillations of the Earth's magnetic field
e) [x] atopy
--------------------------------------------------------------------
91. CM Contributing factors of intrinsic asthma are:

a) [x] Smoking
b) [x] Air pollution
c) [ ] Koch's bacillus
d) [x] Influenza viruses and adenoviruses
e) [ ] Dermatophagoides pteronyssinus
---------------------------------------------------------------------
92. CM Select the pathogenetic mechanism of allergic asthma

a) [ ] type IV hypersensitivity reaction


b) [ ] type V hypersensitivity reaction
c) [ ] type II hypersensitivity reaction
d) [x] type I hypersensitivity reaction
e) [x] type III hypersensitivity reaction
---------------------------------------------------------------------
93. CM Select the bronchial changes consistent with an asthma attack:

a) [x] small airways narrowing


b) [ ] spasm of large airways
c) [x] edema of the bronchial mucosa
d) [ ] bronchial infection
e) [x] bronchial hypersecretion
---------------------------------------------------------------------
ILD
94. CM Which forms of idiopathic interstitial pneumonitis have acute evolution?

a) [x] cryptogenic organizing pneumonitis


b) [x] acute interstitial pneumonitis
c) [ ] nonspecific interstitial pneumonitis
d) [ ] usual interstitial pneumonitis/ Idiopathic Pulmonary Fibrosis
e) [ ] desquamative interstitial pneumonitis
---------------------------------------------------------------------
CM Which form of idiopathic interstitial pneumonitis have chronic evolution ?

a) [ ] cryptogenic organizing pneumonitis


b) [ ] acute interstitial pneumonitis
c) [x] nonspecific interstitial pneumonitis
d) [x] usual interstitial pneumonitis/ Idiopathic pulmonary fibrosis
e) [ ] desquamative interstitial pneumonitis
---------------------------------------------------------------------
95. CS Cysts, fibroblast foci, alternating areas of normal lung parenchyma and areas of fibrosis in a lung
biopsy specimen characterize:

a) [ ] Lung abscess
b) [ ] Pneumonia
c) [x] Idiopathic pulmonary fibrosis
d) [ ] Nonspecific interstitial pneumonitis
e) [ ] Lung cancer
---------------------------------------------------------------------
96. CS On HRCT were determined subpleural honeycombing with lower lobe predominance, widespread
diffuse reticular opacities, ground glass opacities in restricted areas, traction. What is the most likely
diagnosis?

a) [ ] COPD
b) [ ] Bronchiectasis
c) [ ] Cystic fibrosis
d) [x] Idiopathic pulmonary fibrosis
e) [ ] Sarcoidosis
---------------------------------------------------------------------
97. CM Exacerbation of idiopathic pulmonary fibrosis should be suspected in the following cases:

a) [ ] aggravation of dyspnea over the last year


b) [x] appearance of "ground glass" opacities with radiological progression
c) [x] aggravation of dyspnea in the last month
d) [ ] appearance of bilateral infiltrative opacities with radiological progression
e) [ ] association of an infection
---------------------------------------------------------------------
98. CM Which form of idiopathic interstitial pneumonitis are smoking related?

a) [ ] Cryptogenic organizing pneumonitis


b) [ ] Acute interstitial pneumonitis
c) [x] Respiratory bronchiolitis - ILD
d) [ ] Usual interstitial pneumonitis/ Idiopathic Pulmonary Fibrosis
e) [x] Desquamative interstitial pneumonitis
---------------------------------------------------------------------
99. CM What are the clinical manifestations consistent with idiopathic pulmonary fibrosis?

a) [ ] fever
b) [x] progressive dyspnea
c) [ ] dyspnea with an acute onset
d) [ ] cough
e) [x] velcro crackles
-------------------------------------------------------------------
100. CM What imaging signs can be determined on a chest X-ray in a patient with idiopathic pulmonary
fibrosis?

a) [x] linear opacities


b) [ ] cavities
c) [x] reticular opacities
d) [ ] pneumothorax
e) [x] honeycombing
---------------------------------------------------------------------
101. CM What imaging signs can be determined on chest HRCT in a patient with idiopathic pulmonary
fibrosis?

a) [x] traction bronchiectasis


b) [ ] cavities
c) [x] reticular opacities
d) [x] minimal „ground glass" opacities
e) [x] honeycombing
---------------------------------------------------------------------
102. CM Name investigations used in the diagnosis of idiopathic pulmonary fibrosis
a) [x] chest X-ray
b) [x] chest HRCT
c) [ ] pulmonary scintigraphy
d) [x] pulmonary function tests and DLCO
e) [ ] skin tests
---------------------------------------------------------------------
103. CM The most important complications of idiopathic pulmonary fibrosis are:

a) [x] infections (Pneumocystis jiroveci, Aspergillus)


b) [ ] lung abscess
c) [ ] pulmonary gangrene
d) [x] lung cancer
e) [x] rapid deterioration
---------------------------------------------------------------------
104. CM Morphological signs characteristic for hystological pattern of usual interstitial pneumonitis (UIP) are:

a) [ ] granulomas
b) [x] fibroblast foci
c) [x] thickening of the alveolar septa
d) [x] hyperplasia of type II pneumocytes
e) [ ] the presence of eosinophils in the intraseptal cellular infiltrate
---------------------------------------------------------------------
105. CM In normal lung regeneration, repair process is completed by:

a) [ ] the predominance of epithelial cell apoptosis


b) [ ] reduced myofibroblasts apoptosis
c) [ ] secretion of surfactant
d) [x] the predominance of myofibroblasts apoptosis
e) [x] proliferation and differentiation of epithelial cells
--------------------------------------------------------------------
106. CM Markers that correlate with the severity of lung injury are:

a) [x] ESR
b) [x] Fibrinogen
c) [x] LDH
d) [x] C-reactive protein
e) [ ] CFK
---------------------------------------------------------------------
107. CM In case new areas of „ground glass" opacity and / or areas of pulmonary consolidation on HRCT in a
patient with idiopathic pulmonary fibrosis we suspect:

a) [ ] COPD
b) [x] a superimposed infection
c) [x] exacerbation of idiopathic pulmonary fibrosis
d) [ ] development of pneumothorax
e) [ ] development of piopneumothorax
---------------------------------------------------------------------
108. CM Bronchoalveolar lavage (BAL) in idiopathic pulmonary fibrosis will present:

a) [x] Eosinophils > 20%


b) [ ] Lymphocytosis >15%
c) [x] Neutrophils > 5%
d) [ ] Eosinophils under 5%
e) [ ] Pigmented macrophages >20%
---------------------------------------------------------------------
109. CM The following are major criteria for the diagnosis of idiopathic pulmonary fibrosis in the absence of
lung biopsy, except:

a) [x] restriction (reduced FVC with increased FVC 1/FVC) and impaired gas exchange
b) [ ] age over 50 years
c) [ ] restriction (increased FVC with reduced FVC 1/FVC ratio) and impaired gas exchange
d) [x] bilateral basal irregular linear and reticular opacities on HRCT
e) [x] exclusion of other known causes of diffuse interstitial lung disease
---------------------------------------------------------------------
110. CM Lymphocytosis in BAL (bronchoalveolar lavage) is suggestive for:

a) [x] Sarcoidosis
b) [x] Cryptogenic organizing pneumonitis
c) [ ] Idiopathic pulmonary fibrosis
d) [x] Nonspecific interstitial pneumonitis
e) [ ] Acute interstitial pneumonitis
---------------------------------------------------------------------
111. CM Drugs used in the treatment of idiopathic pulmonary fibrosis are:

a) [x] Pirfenidone
b) [ ] Cocarboxylase
c) [x] Nintedanib
d) [ ] Adrenaline
e) [ ] Prednisolon
---------------------------------------------------------------------
112. CS Subpleural "honeycombing" on HRCT is in suggestive of:

a) [ ] Sarcoidosis
b) [ ] COPD
c) [ ] Pulmonary infection
d) [x] Idiopathic pulmonary fibrosis
e) [ ] Cryptogenic organizing pneumonitis
---------------------------------------------------------------------
113. CM 72-year-old patient, smoker 40 pack /year, complains of an exhausting dry cough, dyspnea at rest,
marked fatigue, general weakness. The onset of the disease was insidious, with progressive aggravation of
dyspnea and cough during the past year. Physical exam: acrocyanosis, clubbing, emphysematous chest, on
lung percussion - hyperresonant sound (box-like), bilateral subscapular dullness, auscultation - decreased
vesicular murmur, bilateral subscapular crepitations. RR 26/min, HR 110/min, BP 130/70 mmHg. SaO2
90%. On chest X-ray - interstitial syndrome. What are the investigations required for diagnosis?

a) [x] Spirometry
b) [x] DLco
c) [ ] Pulmonary scintigraphy
d) [ ] ECG
e) [x] Chest HRCT
PLEURISY
114. CS Dry pleuritis is frequently associated with the following conditions, except:

a) [ ] Tuberculosis
b) [ ] Rheumatoid arthritis
c) [x] Heart failure
d) [ ] Sarcoidosis
e) [ ] Uremia
--------------------------------------------------------------------
115. CS Name features that are not characteristic for the pain in dry pleuritis:
a) [ ] amplification at deep breathing, cough
b) [ ] sharp, „stabbing" character
c) [ ] relief at limitation of thoracic movements
d) [x] subsides at nitroglycerin administration
e) [ ] similar to the pain of rib fractures
---------------------------------------------------------------------
116. CS The signs listed below are typical for dry pleuritis, except:

a) [ ] rapid shallow breathing with limited ipsilateral respiratory movements


b) [ ] on palpation- limitation of thoracic movements and sometimes perception of pleural friction
c) [ ] frequently the patient is lying on the affected side to limit chest expansion
d) [ ] on auscultation - pleural friction
e) [x] on percussion- sound of a "falling drop"
---------------------------------------------------------------------
CM Common causes of transudates are the following, except:

a) [ ] Congestive heart failure


b) [ ] Hepatic cirrhosis
c) [ ] Nephrotic syndrome
d) [x] Pleural empyema
e) [ ] Hypoalbuminemia
---------------------------------------------------------------------
117. CM Indications for diagnostic thoracentesis in a patient with heart failure and pleural effusion are the
following, except:

a) [x] disappearance of pleural effusion after diuretic therapy


b) [ ] febrile patient with pleural effusion
c) [ ] pleural effusion with pleuritic chest pain
d) [ ] unilateral pleural effusion
e) [ ] persistent pleural effusion despite diuretic therapy
--------------------------------------------------------------------
CM The following are not features of transudates caused by liver cirrhosis:

a) [ ] translocation of peritoneal fluid to the pleural space through the diaphragm


b) [x] commonly located on the left side
c) [ ] commonly located on the right side
d) [ ] contribution of hypoproteinemia
e) [ ] contribution of decreased colloid osmotic pressure
---------------------------------------------------------------------
118. CS Main mechanisms of development of pleural exudates are the following, except:

a) [ ] extension to the pleura of the lung diseases


b) [ ] extension to the pleura of diseases of mediastinum and abdominal organs
c) [x] extravasations of fluid in the pleural space in case of congestive heart failure
d) [ ] pleural involvement form systemic diseases
e) [ ] primary lesion of the pleura
---------------------------------------------------------------------
119. CS Causes of exudates are the following, except:
a) [ ] Pneumonia
b) [ ] Malignancies
c) [x] Congestive heart failure
d) [ ] Collagen diseases
e) [ ] Abdominal abscess (subphrenic, hepatic, splenic)
---------------------------------------------------------------------
120. CS Dyspnea from pleural effusion can be explained by the following mechanisms, except:
a) [ ] Lung compression by the accumulated fluid
b) [ ] Ventilation disorders as a result of lung compression
c) [ ] Perfusion disorders as a result of lung compression
d) [x] Bronchial hyperreactivity with consequent obstruction
e) [ ] Disruption in diaphragmatic mechanics as a result of fluid accumulation in the pleural cavity
---------------------------------------------------------------------
121. CS Radiologically, minimal pleurisy is manifested by:

a) [ ] Opacity (usually with supracostal intensity) with concave upper limit like Damoiseau line
b) [x] Amputation of costophrenicus sinus; sometimes - discrete elevation of the diaphragm with decreased
amplitude of movement
c) [ ] Opacity over the whole hemithorax
d) [ ] Contralateral mediastinal displacement with diaphragm descending
e) [ ] Hydropneumotorax on the left, mediastinal and subcutaneous emphysema
--------------------------------------------------------------------
122. CS What are the radiological signs of a moderate pleural effusion?

a) [ ] Hydropneumotorax on the left, mediastinal and subcutaneous emphysema


b) [ ] Amputation of costophrenicus sinus; sometimes - discrete elevation of the diaphragm with decreased
amplitude of movement
c) [x] Opacity (usually of supracostal density) with concave upper limit like Damoiseau line, which changes with the
change of the patient's position
d) [ ] Opacity over the whole hemithorax
e) [ ] Contralateral mediastinal displacement with diaphragm descending
---------------------------------------------------------------------
123. CSThe most sensitive test in detecting small effusions is the:

a) [ ] Posterior-anterior chest X-ray


b) [ ] Lateral chest X-ray
c) [ ] Conventional tomography
d) [x] Ultrasound examination of the chest
e) [ ] Computed tomography
---------------------------------------------------------------------
124. CS In a pleural effusion computed tomography has the decisive role in:

a) [ ] Detecting small effusions


b) [ ] Pleural puncture guidance
c) [ ] Assessment of the amount of liquid
d) [ ] Suggesting the appearance of pleural fluid (hemothorax, empyema etc.)
e) [x] Specifying the parenchymal condition in the area of pleural opacity (lung abscess, pneumonia, bronchogenic
cancer)
---------------------------------------------------------------------
125. CS Select macroscopic description of pleural fluid which corresponds to a chylothorax or
pseudochilotorax:

a) [ ] Colorless liquid, clear, odorless


b) [ ] Purulent liquid with unpleasant odor
c) [x] Opalescent and milky liquid
d) [ ] Hemorrhagic, pink-colored, foamy fluid
e) [ ] Viscous, cloudy, odorless
--------------------------------------------------------------------
126. CS What biochemical test is characteristic to pleural effusion due to mesothelioma:

a) [ ] pleural fluid glucose value is equal to that of serum


b) [ ] significant increase in amylase levels in pleural fluid
c) [ ] increased levels of triglycerides and fatty acids
d) [ ] increased level of cholesterol in pleural effusion
e) [x] increased levels of hyaluronic acid in pleural effusion
---------------------------------------------------------------------
127. CS In which disease significantly increases pleural amylase?

a) [x] Pleurisy from pancreatitis or pancreatic pseudocyst


b) [ ] Parapneumonic pleurisy
c) [ ] Pleurisy as a result of thoracic lymph duct rupture
d) [ ] Pleurisy in rheumatoid arthritis
e) [ ] Pleural effusion in congestive heart failure
---------------------------------------------------------------------
128. CS WBC less than 1000/mm3 in cytology examination of pleural fluid collections is typical to:

a) [ ] parapneumonic effusion
b) [ ] tuberculous effusion
c) [x] transudate from congestive heart failure
d) [ ] neoplastic exudates
e) [ ] exudate from collagen disorders
---------------------------------------------------------------------
129. CS The examination of pleural fluid of a 28-year-old patient, revealed sero-citrine exudate, with
predominantely lymphocytes in cytological examination and present AFB in bacteriologic examination.
What is the probable diagnosis?

a) [ ] parapneumonic effusion
b) [ ] neoplastic exudates
c) [ ] transudate from congestive heart failure
d) [ ] exudate from the obstruction of thoracic lymph duct
e) [x] tuberculous effusion
---------------------------------------------------------------------
130. CS The examination of 68-year-old patient pleural fluid, revealed a hemorrhagic sero-citrine exudate, with
predominantly mesothelial cells, lymphocytes and atypical cells in cytological examination. Pleural biopsy
was taken to confirm the diagnosis. What is the probable diagnosis?

a) [ ] parapneumonic effusion
b) [x] malignant exudate
c) [ ] transudate from congestive heart failure
d) [ ] exudate from the obstruction of thoracic lymph duct
e) [ ] tuberculous effusion
---------------------------------------------------------------------
131. CS The examination of a 70-year-old patient pleural fluid, revealed sero-citrine liquid, the ratio LDH
pleural / serum LDH <0,6 and small quantity of mesothelial cells in cytological examination. What is the
probable diagnosis?

a) [ ] parapneumonic effusion
b) [ ] malignant exudates
c) [x] transudate from congestive heart failure
d) [ ] exudate from the obstruction of thoracic lymph duct
e) [ ] tuberculous effusion
---------------------------------------------------------------------
132. CS The exam of pleural fluid in a 42-year-old patient, revealed a milky exudate, with increased levels of
triglycerides and fatty acids, low cholesterol concentration. What is the probable diagnosis?

a) [ ] parapneumonic effusion
b) [ ] neoplastic exudate

c) [ ] transudate from congestive heart failure

d) [x] exudate from the obstruction of thoracic lymph duct

e) [ ] tuberculous effusion
---------------------------------------------------------------------
133. CS A 36-year-old patient in a few days after the onset of pneumonia, despite antibacterial treatment, has
persisting fever, chest pain and coughing have intensified. Objective: besides the congestion syndrome, the
abolition of vocal vibrations in the lower left hemithorax has been detected, dullness, pleuretic rub.
Radiologically - homogenous opacity on a background of lung consolidation. Examination of pleural fluid
shows a citrine effusion with a large amount of fibrin, cytology - numerous intact polymorphnuclear
neutrophils. What complication of pneumonia is found in this patient?

a) [ ] congestive heart failure with pleural transudate

b) [ ] pleural empyema

c) [x] parapneumonic pleurisy

d) [ ] pyopneumothorax

e) [ ] purulent pericarditis
---------------------------------------------------------------------
134. CS Pneumonia in a 36-year-old patient got complicated with serofibrinous (aseptic) pleurisy. What drugs
can be associated to the etiologic antibiotic treatment of pneumonia?

a) [ ] inhaled в2-agonists

b) [ ] inhaled corticosteroids

c) [ ] expectorants

d) [x] NSAIDs

e) [ ] antiviral drugs
---------------------------------------------------------------------
135. CS An obese 58-year-old patient has undergone a surgery for phlebothrombosis of his right leg 2 weeks
ago. After immobilization for 10 days he started complaining of dyspnea, small hemoptysis. On the chest x-
ray - radiographic changes characteristic to pulmonary infarction with pleural effusion on the left. Pleural
fluid examination shows a predomination of serohemorrhagic exudate with erythrocytes and a moderate
amount of polymorphonuclear cells. What is the probable diagnosis?

a) [ ] parapneumonic exudate

b) [ ] neoplastic exudate

c) [ ] exudate in the obstruction of thoracic lymph duct

d) [ ] tuberculous exudate

e) [x] exudate of pulmonary thromboembolism


---------------------------------------------------------------------
136. CM In which exudates can be seen leukopenia?
a) [x] Viral pleurisy

b) [ ] Pneumonia with pleurisy

c) [x] Pleural effusion in systemic lupus erythematosus

d) [ ] Subphrenic abscess with pleurisy

e) [ ] Rupture of the esophagus (mediastinitis and empyema)


---------------------------------------------------------------------
137. CM In which exudates can be seen leukocytosis with left shift?

a) [ ] Viral pleurisy

b) [x] Pneumonia with pleurisy

c) [ ] Pleural effusion in systemic lupus erythematosus

d) [x] Subphrenic abscess with pleurisy

e) [x] Rupture of the esophagus (mediastinitis and empyema)


---------------------------------------------------------------------
138. CM Select radiological signs of a massive effusion:

a) [ ] Hidropneumotorax on the left, mediastinal and subcutaneous emphysema

b) [ ] Amputation of sinus costophrenicus; sometimes - discrete ascension of the diaphragm with decrease in its
excursion
c) [x] Opacity over the whole hemithorax

d) [x] Contralateral mediastinal displacement with diaphragm descending

e) [ ] Opacity (usually of supracostal intensity) with concave upper limit in Damoiseau line
---------------------------------------------------------------------
139. CM Select the radiological signs of a minimal pleurisy:

a) [x] Amputation of costophrenic sinus

b) [ ] Opacification of the whole hemithorax

c) [x] Sometimes discrete elevation n of the diaphragm with decrease in its excursion

d) [ ] Opacity with concave upper limit with Damoiseau line

e) [x] In decubitus free fluid forms a strip-like opacity of a medium (subcostal) intensity
---------------------------------------------------------------------
140. CM Ultrasound examination in pleural effusion may be useful for:

a) [x] Detecting small effusions

b) [x] Pleural puncture guidance

c) [ ] Estimation of lung parenchyma condition behind the area of pleural opacity (pulmonary abscess, pneumonia,
bronhogenic cancer)

d) [x] Determining the amount of fluid


e) [x] Suggesting of the character of pleural fluid (hemothorax, empyema etc.)
--------------------------------------------------------------------
141. CM Dry pleuritis frequently occurs in:

a) [x] Tuberculosis

b) [x] Rheumatoid arthritis

c) [ ] Heart failure

d) [x] Sarcoidosis

e) [x] Uremia
---------------------------------------------------------------------
142. CM Pain in dry pleuritis is characterized by:

a) [x] Amplification at deep breathing, cough

b) [x] Sharp pain

c) [x] Relief at limitation of thoracic movements

d) [ ] Subsides after nitroglycerin administration

e) [ ] Average duration of 3-5 minutes


---------------------------------------------------------------------
143. CM Select the features of the dry pleuritis:

a) [x] Rapid shallow breathing, with limited ipsilateral respiratory movements


b) [x] On palpation-confirmation of respiratory limitation and sometimes perception of pleural friction rub
c) [x] Often the patient is lying on the affected side to limit chest expansion
d) [x] On auscultation - pleural friction
e) [ ] On percussion - the sound of a "falling drop"
---------------------------------------------------------------------
144. CM Select the most frequent causes of transudates:

a) [ ] Pleural empyema

b) [x] Congestive heart failure

c) [ ] Pleural mesothelioma

d) [x] Liver cirrhosis

e) [x] Nephrotic syndrome


---------------------------------------------------------------------
145. CM Select the indications for diagnostic thoracocentesis in a patient with pleural transudate from heart
failure:

a) [ ] The disappearance of pleural effusion after diuretic therapy

b) [x] Febrile patient with pleural effusion

c) [ ] Pleural effusion with pleuretic pain


d) [ ] Unilateral pleural effusion

e) [x] Persistent pleural effusion despite diuretic therapy


------------------------------------------------------------------
146. CM Name features of transude caused by liver cirrhosis:

a) [x] Direct movement of peritoneal fluid to the pleural space through small holes from the diaphragm

b) [ ] Commonly located on the left

c) [x] Commonly located on the right

d) [ ] Hyperbilirubinemia contributes to the development of transudate

e) [x] Hypoproteinemia is the cause of transudate formation due to decreased colloid osmotic pressure
---------------------------------------------------------------------
147. CM Select the most common etiological causes of pleural exudates:

a) [x] Pneumonia

b) [x] Malignancies

c) [x] Collagen diseases

d) [ ] Congestive heart failure

e) [ ] Nephrotic syndrome
---------------------------------------------------------------------
148. CM Name the mechanism of dyspnea in significant pleural effusion:

a) [x] Impairment in diaphragmatic mechanics as a result of fluid accumulation in the pleural cavity
b) [ ] Bronchial hyperreactivity with their consequent obstruction
c) [x] Lung compression by the fluid accumulation
d) [x] Disorders of ventilation and perfusion as a result of lung compression
e) [ ] Main bronchus obstruction by a stretched tumor
--------------------------------------------------------------------
149. CM Select cases of hemorrhagic pleurisy, confirmed by pleural fluid analysis:

a) [x] primary or metastatic pleural cancer

b) [ ] congestive heart failure

c) [x] tuberculosis

d) [x] pleural endometriosis

e) [x] Hodgkin disease


---------------------------------------------------------------------
150. CM Select causes of pleurisy taken by thoracentesis in which opalescent liquid and milky was noted:

a) [ ] Primitive or metastatic pleural cancer

b) [x] Chiliform pleurisy with a lymphatic collection in the pleural cavity

c) [ ] Hodgkin recent hemorrhagic pleurisy


d) [x] Pleurisy with cholesterol effusion in old tuberculosis

e) [x] Chiliform pleurisy with an old lymphatic collection in rheumathiod polyarthritis


---------------------------------------------------------------------
151. CM Select criteria that define a pleural effusion as an exudates

a) [x] positive Rivalta reaction

b) [ ] negative Rivalta reaction

c) [x] pleural LDH / serum LDH ratio ≥0,6

d) [ ] total protein <30 g/l

e) [x] pleural protein / serum protein ratio ≥0,5

Lung abcess
152. CS What is a lung abscess?

a) [ ] diffuse pulmonary suppuration, characterized by the formation of several excavations with 3-5 cm in diameter

b) [ ] diffuse pulmonary suppuration, which is a complication of atypical pneumonia

c) [x] a circumscribed focus of pulmonary suppuration, which may be a complication of pneumonia with anaerobic
germs

d) [ ] a circumscribed focus of pulmonary suppuration, which is a mandatory and permanent complication of


atypical pneumonia

e) [ ] diffuse pulmonary suppuration, characterized by necrotic lesions and lack of delimitation of the inflammation
---------------------------------------------------------------------
153. CS Clinical picture in bronchiectasis is characterized by:

a) [x] chronic cough with a expectoration of smelly sputum

b) [ ] chronic expectoration of rusty sputum

c) [ ] chronic cough without sputum

d) [ ] chronic cough, with vomica

e) [ ] chronic cough with expectoration of pearly sputum


---------------------------------------------------------------------
154. CS The best imaging method for detection of bronchiectasis:

a) [ ] chest radiography

b) [ ] bronchography

c) [ ] bronchoscopy

d) [ ] pulmonary scintigraphy

e) [x] high-resolution computed tomography


---------------------------------------------------------------------
155. CSThe most probable etiology of lung abscess is the following, except:

a) [ ] oral anaerobes

b) [ ] Enterobacteriaceae

c) [x] Mycoplasma pneumoniae

d) [ ] Klebsiella pneumoniae

e) [ ] Staphylococcus aureus
---------------------------------------------------------------------
156. CS A 39 years old patient, complains of cough with expectoration of purulent sputum in an amount of 300-
400 ml/24h, periodical haemoptysis, weight loss, fever (39 degrees C). Physical examination: pale skin,
RR=36/ min, dullness at right axillary region, amphoric breathing, bullous rales. Laboratory examination:
CBC: Hb-9dg/dl, ESR 50 mm / h, WBC - 18 x109/l , nonsegmented - 32%, metamyelocytes -2%, serum
protein - 43 g/l. Chest Xray - cavitary syndrome. Definitive diagnosis requires following investigations
except:
a) [ ] chest HRCT

b) [ ] lung scintigraphy

c) [x] skin test allergens

d) [ ] AFB test of sputum and atypical cells

e) [ ] spirography
---------------------------------------------------------------------
157. CS Name the entry routes of germs into the lungs in pulmonary suppurations:

a) [ ] macroaspiraţion

b) [ ] hematogenous dissemination

c) [ ] lymphogenous dissemination

d) [ ] thoracic trauma

e) [x] all the mentioned routes


---------------------------------------------------------------------
158. CS Secondary lung abscess:

a) [ ] is a complication of atypical pneumonia

b) [ ] occurs on previously healthy lung tissue

c) [x] is a complication of an existing local damage (cancer, bronchial stenosis, etc.)

d) [ ] is an natural evolution of a pneumonia with certain bacteria (Staphylococcus, Klebsiella, Pseudomonas)

e) [ ] there is no such thing as "secondary lung abscess"


---------------------------------------------------------------------
159. CS A 25 years old patient complains from his childhood of muco-purulent cough with moderate
expectoration. He mentions exacerbations of the disease 2 times a year, the sputum becomes purulent,
mainly expectorated in the morning, or when he changes body position, in amounts of 250 ml during a day.
Auscultation - bullous rales. Lab: CBC - neutrophil leukocytosis, ESR 30 mm / h. Sputum microscopy -
100% neutrophils. What is the next investigation to confirm the suggested diagnosis?
a) [ ] Bronchoscopy
b) [x] HRCT
c) [ ] Chest X-ray
d) [ ] Lung scintigraphy
e) [ ] Spirometry
-------------------------------------------------------------------
160. CM Name the factors necessary for the formation of the primitive lung abscess:

a) [ ] pulmonary infection

b) [ ] infection of the bronchial tree

c) [x] gingival infection

d) [x] aspiration

e) [ ] lung surgery
---------------------------------------------------------------------
161. CM Name the radiological changes that can be observed in a lung abscess:

a) [ ] "honeycomb" image

b) [x] round opacity or a pneumonia like image

c) [x] air-fluid level

d) [ ] "soap bubbles" images

e) [ ] "signet ring" image


---------------------------------------------------------------------
162. CM List the indications for surgery of lung abscess:

a) [ ] all cases of acute pulmonary abscess

b) [x] all patients with lung abscess, in which conservative treatment failed

c) [x] abscess secondary to a bronchogenic cancer or bronchial stenosis

d) [ ] lung abscess complicated by pleural empyema or pyopneumothorax

e) [x] repeated or risk of massive hemoptysis


---------------------------------------------------------------------
163. CM List the possible causes of bronchiectasis:

a) [ ] disease has unknown cause and can affect anyone

b) [ ] the causes are unknown, but most affected are premature children

c) [x] congenital anomalies of the bronchial wall

d) [x] untreated pertussis in childhood

e) [x] primary ciliary dyskinesia (Kartagener syndrome)


--------------------------------------------------------------------
164. CM What radiological changes are features of bronchiectasis:

a) [x] ringshaped or curvilinear opacities

b) [x] opaque band ("full pipe")

c) [x] "tramtrack"

d) [ ] air bronchogram

e) [x] cystic formations in rosette


---------------------------------------------------------------------
165. CM Bronchiectasis on computed tomography is identified by:

a) [ ] silhouette sign

b) [x] "signet ring" sign

c) [x] "tree in bud"

d) [ ] air alveologram

e) [ ] parahilar homogeneous opacities ("butterfly" sign)


---------------------------------------------------------------------
166. CM Causes of infectious pulmonary cavity are:

a) [x] Pseudomonas aeruginosa

b) [x] Staphylococcus aureus

c) [ ] Streptococcus pneumoniae

d) [x] Klebsiella pneumoniae

e) [ ] Wegener granulomatosis
---------------------------------------------------------------------
167. CM The main routes of entry into the lungs of pathogenic microbial germs that causes pneumonia are:

a) [x] aspiration of organisms that colonize the oropharynx

b) [x] inhalation of infectious aerosols

c) [x] hematogenous dissemination of infection in an extrapulmonary outbreak

d) [x] direct inoculation as a result of tracheal intubation or stab wounds

e) [ ] lymphatic spread of infection in an extrapulmonary outbreak


---------------------------------------------------------------------
168. CM Which of the following are non-infectious causes of lung cavity:

a) [ ] diabetes

b) [ ] renal disease

c) [ ] sarcoidosis
d) [x] lung cancer

e) [x] Wegener granulomatosis


---------------------------------------------------------------------
169. CM The indications for surgery of lung abscess are:

a) [ ] cough

b) [ ] nocturnal dyspnea

c) [x] malignancy

d) [ ] atelectasis

e) [x] massive hemoptysis


---------------------------------------------------------------------
170. CM Anaerobic pneumonia can be treated with:

a) [x] combination of metronidazole, penicillin G

b) [x] clindamycin

c) [ ] metronidazole is appropriate for gram-positive microaerophilic cocci

d) [ ] metronidazole should be supplemented with a beta-lactam agent

e) [x] penicillin G
--------------------------------------------------------------------
171. CM The following infectious agents are rarely causes of lung abscess:

a) [x] H. influenzae

b) [ ] aerobic gram-negative enteric bacilli

c) [x] Legionella

d) [x] Mycoplasma pneumoniae

e) [x] Histoplasma capsulatum


---------------------------------------------------------------------
172. CM Establishing the diagnosis of anaerobic pulmonary infection is ensured by culturing the following
specimens:

a) [ ] sputum

b) [x] secretion obtained by from transtracheal aspiration

c) [x] the material obtained by transthoracic lung puncture

d) [x] obtained by brushing during bronchoscopy

e) [ ] oropharyngeal swab
--------------------------------------------------------------------
173. CM Anaerobic pneumonia can be treated with:

a) [ ] gentamicin
b) [x] clindamycin

c) [ ] tobramycin

d) [ ] gentamicin + metronidazole

e) [x] metronidazole in combination with penicillin G


---------------------------------------------------------------------
174. CM Pulmonary cavity can be produced by:

a) [x] M. tuberculosis

b) [ ] M. pneumoniae

c) [x] Neoplasms

d) [ ] Virus

e) [x] Wegener granulomatosis


---------------------------------------------------------------------
175. CM The most frequent agents involved in the development of pulmonary abscess and gangrene are:

a) [ ] Legionella pneumoniae

b) [ ] Mycoplasma pneumoniae

c) [x] Klebsiella pneumoniae

d) [x] Bacterioides

e) [x] Staphylococcus aureus


---------------------------------------------------------------------
176. CM Radiological differential diagnosis of lung abscess is performed with:

a) [x] infected echinoccocal cyst

b) [x] aspergilloma

c) [x] cavitary pulmonary tuberculosis

d) [ ] Hodjkin lymphoma

e) [x] cavitary form of lung cancer


---------------------------------------------------------------------
177. CM Complications of acute pulmonary abscess are:

a) [x] pleural empyema

b) [ ] diffuse emphysema

c) [ ] asthma

d) [x] pulmonary hemorrhage

e) [x] sepsis
--------------------------------------------------------------------
178. CM Which of the following conditions increase the risk of lung suppuration?

a) [ ] high blood pressure

b) [x] diabetes

c) [x] advanced lung cancer

d) [ ] gastroduodenal ulcers

e) [x] prolonged corticosteroid


---------------------------------------------------------------------
179. CS A 30 years old patient is suffering from lung abscess for 4 weeks. Last two days his condition worsened
due to increased dyspnea, chills, fever. Radiological appearance reveals pleural empyema. The most
appropriate treatment is:

a) [ ] high doses of antibiotics

b) [ ] pleural lavage

c) [x] surgical treatment by draining the pleural cavity

d) [ ] mucolytics

e) [ ] physiotherapy treatment
---------------------------------------------------------------------
180. CM Which pathology is characterized by increased triglycerides and fatty acids in the pleural fluid?

a) [ ] Pleurisy in pancreatitis or pancreatic pseudocyst

b) [ ] Pleurisy parapneumonica

c) [x] Pleurisy in thoracic lymph duct rupture

d) [x] Pleurisy in obstruction of thoracic duct lymph

e) [ ] Pleural effusion in congestive heart failure


---------------------------------------------------------------------
181. CS Definition of a pulmonary gangrene:
a) [ ] diffuse pulmonary suppuration caused by aerobes and characterized by formation of a number of small
excavations in a field of pneumonia

b) [ ] pulmonary suppuration, caused by anaerobes, evolves towards the formation of a circumscribed focus in a
field of pneumonia

c) [x] diffuse pulmonary suppuration, characterized by necrotic lesions and lack of delimitation of the inflammatory
sites

d) [ ] well defined pulmonary suppuration with expectoration of a necrotized fetid sputum

e) [ ] diffuse pulmonary suppuration called necrotizing pneumonia


---------------------------------------------------------------------

182. CS A 45 years old smoker, has undergone a staphylococcal pneumonia. After 2 weeks of clinical
improvement, the general condition worsened again with high fever - 40°C, preceded by chills, chest pain,
and cough with grey sputum in small amounts with foul odor. Chest X ray indicate the presence of well-
defined round opacity in the right lower lobe. The most likely diagnosis is:

a) [ ] bronchopneumonia,

b) [ ] bronchiectasis

c) [x] acute pulmonary abscess

d) [ ] pulmonary tuberculosis (infiltrative form)

e) [ ] lobar pneumonia (pneumococcal)


---------------------------------------------------------------------
183. CS A 40-year-old man, smoker, 6 months ago suffered from acute pulmonary abscess. Manifested by:
cough with muco-purulent expectoration (100-200 ml/24h), episodic - low grade fever, fatigue. pale-skin,
malnutrition, drumstick fingers . Lung auscultation revealed an amphoric sound, bullous rales over the right
middle lobe. Which of listed the diseases is most probable?

a) [ ] bronchiectasis

b) [ ] fungal pneumonia with abscess formation

c) [x] chronic lung abscess

d) [ ] pulmonary tuberculosis (cavitary form)

e) [ ] lobar pneumonia (pneumococcal) with abscess formation


--------------------------------------------------------------------
184. CS A 41 year old patient, 3 months ago suffered an acute respiratory illness, followed by chills, fever,
cough. He did not receive any treatment. During this period, he noted an increased amount of sputum, with
bad odor, profuse sweating, low grade fever, dyspnea, weight loss. Objectives: tympanic sound over the left
lower lobe, bulous rales. Chest X ray - cavity with air-fluid level in the left lower lobe accompanied by
with intense infiltration of adjacent lung tissue. The diagnosis is:

a) [ ] acute pneumonia

b) [ ] pleural empyema

c) [ ] infected lung cyst

d) [x] lung abscess

e) [ ] lung cancer
---------------------------------------------------------------------
185. CS Sputum in lung suppurations has the following characteristics, except:

a) [ ] purulent sputum, yellow-green, covered with a thin layer of foamy serous fluid

b) [ ] bloody purulent sputum

c) [ ] multilayered sputum

d) [ ] purulent sputum, blood regularly-stained

e) [x] foamy pinky sputum


--------------------------------------------------------------------
186. CS Radiological aspect of pulmonary gangrene is characterized by:

a) [ ] defined homogenous opacity

b) [x] polisegmented poor defined opacities

c) [ ] paramediastinal round opacity

d) [ ] triangular opacity with the base towards the parietal pleura

e) [ ] thin-walled round cavity


Bronchiectasis
187. CS Bronchiectasis is:

a) [ ] intermittent and reversible dilatation of the subsegmental bronchi

b) [ ] intermittent dilatation of the large airways

c) [ ] permanent dilation of the terminal bronchioles and alveoli

d) [x] permanent and irreversible dilatation of the subsegmental bronchi

e) [ ] any of the above is incorrect


---------------------------------------------------------------------

LUNG CANCER

188. CS Which of the listed are the possible sites of metastases from lung cancer to multiple locations, except:

a) [ ] brain

b) [ ] bone

c) [ ] liver

d) [ ] medullary

e) [x] spleen
--------------------------------------------------------------------
189. CS The following events are secondary to endobronchial lung tumor development, except:

a) [x] whooping cough

b) [ ] hemoptysis

c) [ ] stridor

d) [ ] pneumonia

e) [ ] pleural pain
--------------------------------------------------------------------
190. CS sec What are the ages of maximal incidence of lung cancer:

a) [ ] 30-35
b) [x] 45-60

c) [ ] 65-75

d) [ ] 25 - 35

e) [ ] 75 years
---------------------------------------------------------------------
191. CS How much is smoking increasing the risk of lung cancer:

a) [ ] 13 times

b) [ ] from 2 to 3 times

c) [x] 20-fold

d) [ ] 50 times

e) [ ] the risk is not different to that of a non-smoker


---------------------------------------------------------------------
192. CS What is the most common histological type of lung cancer in those who never smoked, in women and
in younger patients:

a) [ ] small cell carcinoma

b) [ ] large cell carcinoma (anaplastic)

c) [ ] carcinoid tumors

d) [x] adenocarcinoma

e) [ ] epidermoid carcinoma
---------------------------------------------------------------------
193. CM Contraindications to surgery in lung cancer are:

a) [x] DLCO < 50%

b) [ ] FEV1> 2.5 l

c) [ ] FEV1 from 1.5 to 2.4 l

d) [x] FEV1 <1.5 l

e) [ ] mild pulmonary hype


---------------------------------------------------------------------

194. CM What are the examinations required for histologic diagnosis of malignancy in lung cancer:

a) [ ] Chest X-ray, chest computed tomography, radiography of the suspicious bones

b) [x] bronchial or transbronchial biopsy with bronchoscope, biopsy of accessible peripheral lesions, percutaneous
biopsy of a lymph node

c) [ ] fiber-optic bronchoscopy, lung ventilatory tests, coagulation tests


d) [x] bone marrow aspiration or biopsy, diagnostic thoracentesis, mediastinoscopy with lymph node biopsy

e) [ ] complete history, complete blood count, chest radiography, chest computed tomography
---------------------------------------------------------------------
195. CM The following events characterize Horner syndrome:

a) [ ] exophthalmos

b) [x] ptosis

c) [x] miosis

d) [x] ipsilateral loss of sweating

e) [ ] transpiration loss contralateral


---------------------------------------------------------------------
196. CM Symptoms and signs associated to the endobronchial growth of lung cancers:

a) [x] hemoptysis
b) [ ] pain
c) [x] cough
d) [x] wheezing
e) [ ] hoarseness
---------------------------------------------------------------------
197. CM The occurrence of radiation pneumonitis after radiotherapy in patients with lung cancer is directly
proportional to:

a) [ ] the presence of metastases

b) [ ] tumor histology

c) [x] the radiation dose

d) [x] lung volume included in the radiation field

e) [ ] patient age
---------------------------------------------------------------------
198. CM Collection of histologic samples for diagnosis of lung cancer can be performed by:

a) [x] bronchial or transbronchial biopsy with fiber optic bronchoscope

b) [x] lymph node biopsy at mediastinoscopy

c) [x] during tumor resection surgery

d) [x] percutaneous biopsy of an enlarged lymph node

e) [ ] liver or bone scintigraphy


---------------------------------------------------------------------
199. CM Pancoast syndrome is characterized by:

a) [ ] coexistence of Horner syndrome


b) [x] apical localization of the lung cancer
c) [ ] lower lobe localization of the lung cancer
d) [x] shoulder pain
e) [ ] radiological signs of VII-VIII rib destruction
PNEUMONIA
200. CM "Typical" pneumonic syndrome (sudden onset of productive cough with purulent sputum, pleural
chest pain, signs of pulmonary consolidation) is usually caused by:

a) [x] Streptococcus pneumoniae

b) [ ] Mycoplasma pneumoniae

c) [ ] Pneumocystis jiroveci

d) [ ] Coxiella burnetii

e) [ ] Chlamydophila psittaci
---------------------------------------------------------------------
CS Germs that cause lung abscess can be the following, except:

a) [ ] anaerobes

b) [ ] Gram-negative enteric bacilli

c) [x] Mycoplasma pneumoniae

d) [ ] Streptococcus pneumoniae type III

e) [ ] Staphylococcus aureus
---------------------------------------------------------------------
201. CS sec "Atypical" pneumonia is usually determined by:

a) [x] Mycoplasma pneumoniae

b) [ ] Streptococcus pneumoniae

c) [ ] Klebsiella pneumoniae

d) [ ] Pseudomonas aeruginosa

e) [ ] aerobic Gram-negative enteric bacilli


---------------------------------------------------------------------
202. CS Pneumonia in patients with intravenous catheters is most commonly caused by:

a) [x] Staphylococcus aureus

b) [ ] Mycoplasma pneumoniae

c) [ ] Pseudomonas aeruginosa

d) [ ] Oral Anaerobes

e) [ ] Haemophylus influenzae
--------------------------------------------------------------------
203. CS Which of the following cannot be cultured by routine methods (regular culture medium):

a) [ ] Mycoplasma pneumoniae

b) [ ] Chlamydophila pneumoniae
c) [ ] Pneumocystis jiroveci

d) [ ] Mycobacteria

e) [x] any of the mentioned germs


---------------------------------------------------------------------
204. CS The drug of choice in Legionella pneumophila pneumonia is:

a) [ ] Ampicillin

b) [ ] Chloramphenicol

c) [ ] Penicillin

d) [ ] Cephalosporins

e) [x] Azythromycin
---------------------------------------------------------------------
205. CS The following pathogens causing pneumonia cannot be cultured by routine methods (ordinary culture
media), except:
a) [ ] Mycoplasma pneumoniae

b) [ ] Pneumocystis jiroveci

c) [ ] Legionella pneumophila

d) [ ] anaerobic germs

e) [x] Staphylococcus aureus


---------------------------------------------------------------------
206. CS Source of Legionella pneumophila is:

a) [x] the contaminated water

b) [ ] air

c) [ ] soil

d) [ ] food

e) [ ] manure
-------------------------------------------------------------------
207. CS Antibiotics of choice in the treatment of Legionella infection is as follows:

a) [ ] penicillin

b) [x] macrolides

c) [ ] aminoglycosides

d) [ ] cephalosporins

e) [ ] fluorchinolone
---------------------------------------------------------------------
208. CS The drug of choice in the treatment of pneumonia caused by Pneumocystis jiroveci is:
a) [ ] AMO / AC

b) [ ] ceftriaxone

c) [x] trimethoprim-sulfamethoxazole

d) [ ] ceftazidime

e) [ ] amikacin
---------------------------------------------------------------------
209. CS Name the drugs of choice in the treatment of mild community-acquired pneumonia:

a) [ ] clinamycin

b) [ ] III generation cephalosporins

c) [x] macrolides

d) [ ] fluoroquinolones

e) [ ] vancomycin
---------------------------------------------------------------------
210. CS Which pathogen cause lobar pneumonia?

a) [ ] Staphylococcus aureus

b) [ ] Streptococcus viridans

c) [x] Streptococcus pneumoniae

d) [ ] Mycoplasma pneumoniae

e) [ ] influenza viruses
---------------------------------------------------------------------
---------------------------------------------------------------------
211. CS The etiologic treatment of pneumonia caused by Mycoplasma pneumoniae imply the following
antibacterial drugs, except:

a) [x] cephazolin

b) [ ] erythromycin

c) [ ] clarithromycin

d) [ ] doxicycline
e) [ ] azithromycin

--------------------------------------------------------------------
212. CS Complications of pneumococcal pneumonia are the following, except:

a) [ ] sepsis

b) [ ] meningitis

c) [ ] empyema
d) [ ] glomerulonephritis

e) [x] renal amyloidosis


-------------------------------------------------------------------
213. CS A 54 year old patient complained of cough and hemoptysis with an acute onset accompaniend by fever
up to 39.5 °C and chills. Phisical examination: confusion, RR- 30 breaths per minute. Palpation- increased
vocal fremitus; percussion - dullness; auscultation - bronchial breathing, crepitations, pleural friction, HR
100/min; BP - 95/60 mm Hg. Chest Xray - presents a homogeneous opacity of the lower lobe of the right
lung. CBC: WBC - 27.1 x109 / l: nonsegmented - 29%, segmented - 43%, lymphocytes - 22%, monocytes -
6%, ESR - 46 mm / h. The patient was diagnosed with lobar pneumonia. Considering the severity of the
patient, select the possible complications:

a) [ ] destructive lung process

b) [ ] toxic shock

c) [ ] acute respiratory failure

d) [ ] collapse

e) [x] all of the above


--------------------------------------------------------------------
214. CS In patients hospitalized for community acquired pneumonia the most commonly involved pathogen is:

a) [ ] Pseudomonas aeruginosa

b) [x] Streptococcus pneumoniae

c) [ ] Gram-negative enteric bacilli

d) [ ] Haemophilus influenzae

e) [ ] Legionella pneumophila
---------------------------------------------------------------------
215. CS Hospital acquired pneumonia in more than 50% of cases is caused by:

a) [x] Gram-negative bacteria

b) [ ] Mycoplasma pneumoniae

c) [ ] Pseudomonas aeruginosa

d) [ ] Haemophilus influenzae

e) [ ] Chlamydophila pneumoniae
---------------------------------------------------------------------
216. CM Select the statements that are not criteria for hospitalization of patients with pneumonia:

a) [ ] age > 65 years

b) [ ] significant comorbidities

c) [x] fever 38 °C

d) [x] the probable cause of pneumonia - Streptococcus pneumoniae


e) [ ] tachypnea (> 30 breaths / min)
---------------------------------------------------------------------
CM Select the criteria for hospitalization of patients with pneumonia

a) [ ] PaO2 <60 mmHg

b) [ ] PaO2 = 90 mmHg

c) [ ] heart rate 68 beats / minute

d) [x] respiratory rate> 30/min

e) [x] systolic BP <90 mmHg


---------------------------------------------------------------------
CM Empirical oral antimicrobial therapy used in the treatment of ambulatory patients with community-acquired
pneumonia pathogens aims the following germs:

a) [ ] Pseudomonas aeruginosa

b) [x] Mycoplasma pneumoniae

c) [x] Streptococcus pneumonia

d) [x] Chlamydophila pneumoniae

e) [ ] Staphylococcus aureus
---------------------------------------------------------------------
217. CM Pathogens most commonly involved in the etiology of community-acquired pneumonia in old patients
with preexisting chronic respiratory disease are:

a) [ ] Legionella pneumophila

b) [x] Haemophilus influenzae

c) [x] Moraxella catarrhalis

d) [ ] Pneumocystis jiroveci

e) [ ] cytomegalovirus
---------------------------------------------------------------------
COPD AND CHRONIC BRONCHITIS

218. CS The damage of which airways produces sibilant rales on auscultation?


[ ] large bronchi

b) [ ] medium bronchi

c) [x] small bronchi

d) [ ] all the above mentioned

e) [ ] none of the listed


---------------------------------------------------------------------
219. CS What is the most common cause of death of patients with chronic bronchitis?
a) [ ] active inflammatory process in the respiratory system
b) [x] cardio-respiratory failure
c) [ ] thromboembolism of the pulmonary artery
d) [ ] spontaneous pneumothorax
e) [ ] comorbidities
---------------------------------------------------------------------
220. CS When chronic bronchitis requires antibiotics?
a) [ ] during the fall - winter season
b) [ ] as a prophylactic measure 3 times per year
c) [ ] when serous sputum is present
d) [x] when there is purulent sputum
e) [ ] antibiotics are not indicated
---------------------------------------------------------------------
221. CS The main examination method for differential diagnosis of chronic bronchitis from bronchiectasis is:
a) [x] chest high-resolution computed tomography
b) [ ] ChestXR
c) [ ] Bronchoscopy
d) [ ] Bronhography
e) [ ] Spirometry
--------------------------------------------------------------------
222. CS Which of the etiologic factors involved in the pathogenesis of chronic bronchitis is considered to be the
most important:
a) [ ] genetic predisposition
b) [ ] air pollution
c) [x] smoking
d) [ ] occupational exposure to dust and toxic gases
e) [ ] infections
-------------------------------------------------------------------
223. CS Chronic purulent bronchitis is characterized by:
a) [x] persistent or recurrent purulent sputum
b) [ ] pearl sputum

c) [ ] mucous sputum

d) [ ] hemoptysis

e) [ ] wheezing
---------------------------------------------------------------------
224. CS Genetic factors involved in the etiology of chronic bronchitis are:
a) [ ] antithrombin III deficiency

b) [ ] deficiency of complement factors

c) [ ] the lack of transferrin

d) [x] alpha-1 antitrypsin deficiency

e) [ ] excessive activity of leukocyte proteolytic enzymes


---------------------------------------------------------------------
225. CS Chronic bronchitis is defined clinically by cough with expectoration:
a) [ ] at least 2 months / year, more than 2 consecutive years
b) [x] at least 3 months / year, more than 2 consecutive years
c) [ ] at least 6 months / year, more than 3 consecutive years
d) [ ] maximum 3 months / year, 2 years consecutive
e) [ ] chronic bronchitis is defined clinically by continuous cough with expectoration
---------------------------------------------------------------------
226. CM Choose the correct statements about acute bronchitis:
a) [ ] it lasts several months
b) [x ] self-limited disease with complete resolution
c) [x] clinical is manifested by bronchitis syndrome
d) [ ] clinical is manifested by hyperinflation syndrome
e) [ ] radiologicaly is manifested by interstitial syndrome
---------------------------------------------------------------------
227. CS Select the most frequent causes of acute bronchitis:
a) [ ] fungal

b) [ ] bacterial

c) [x] viral

d) [ ] allergic

e) [ ] chemical
---------------------------------------------------------------------
228. CM Choose the correct pathological forms of acute bronchitis:
a) [x] catarrhal
b) [x] hemorrhagic
c) [ ] lymphoproliferative
d) [ ] atrophic
e) [x] ulcerative
---------------------------------------------------------------------
CM Choose the correct statements about acute bronchiolitis:
a) [x] diffuse involvement of small airways
b) [x] the patient presents marked dyspnea

c) [ ] dullness on percussion

d) [ ] is most often caused by atypical agents

e) [x] auscultation - multiple bullous rales


---------------------------------------------------------------------
229. CM Select the antibiotics of choice in acute bronchitis when antibiotic therapy is needed:
a) [x] Amoxicillin

b) [x] Doxycycline

c) [ ] Ceftriaxone

d) [ ] Tobramycin

e) [x] Сefaclor
---------------------------------------------------------------------
230. CM Select correct statements referred to chronic bronchitis:
a) [x] is manifested by cough with expectoration
b) [x] cough is present for at least 3 months per year for more than 2 consecutive years
c) [ ] cough lasts more than 6 months at least 1 year
d) [ ] it is mainly manifested by wheezing
e) [ ] it is a self-limiting condition with complete resolution
--------------------------------------------------------------------
231. CM What are the most common bacteria involved in chronic bronchitis:
a) [ ] Pseudomonas aeruginosa
b) [x] Pneumococcus
c) [x] Haemophilus influenzae

d) [ ] Staphylococcus aureus

e) [ ] Bacteroides
---------------------------------------------------------------------
232. CM Select the clinical forms of chronic bronchitis:
a) [ ] haemorrhagic chronic bronchitis
b) [ ] mixed bronchitis

c) [x] simple chronic bronchitis

d) [x] mucopurulent chronic bronchitis

e) [x] chronic obstructive bronchitis


---------------------------------------------------------------------
233. CM Select the clinical syndromes present in chronic bronchitis:
a) [ ] the pulmonary consolidation syndrome
b) [x] bronchitis syndrome

c) [x] obstructive syndrome

d) [ ] mediastinal syndrome

e) [ ] angina-like syndrome
---------------------------------------------------------------------

234. CM What modifications of spirometric indices are indicative of obstructive respiratory dysfunction:
a) [x] decreased FEV1
b) [ ] increased FEV1
c) [x] decreased FEV1 / FVC
d) [ ] increased FEV1 / FVC
e) [ ] increased PEF
---------------------------------------------------------------------
235. CM What modifications of spirometric indices are not compatible with obstructive respiratory dysfunction:
a) [x] increased FEV1
b) [ ] PEF decreased
c) [x] FEV1 / FVC normal limits
d) [ ] simultaneously decreased FEV1 and FVC
e) [ ] FEF 25-75% decreased
---------------------------------------------------------------------
236. CM Complications of chronic bronchitis include:
a) [x] chronic respiratory failure
b) [ ] pleural empyema
c) [x] cor pulmonale
d) [x] centrolobular emphysema
e) [ ] anemia
---------------------------------------------------------------------
237. CM Complications of chronic bronchitis are all, except:
a) [ ] bronchiectasis
b) [ ] secondary polyglobulia
c) [x] lung abscess
d) [x] pulmonary hydatid cyst
e) [x] idiopatic pulmonary fibrosis
-------------------------------------------------------------------
238. CM The differential diagnosis of chronic bronchitis is made with:
a) [x] Asthma
b) [ ] Upper viral respiratory tract infections
c) [ ] Hysteria
d) [x] Bronchiectasis
e) [x] Chronic sinusitis
---------------------------------------------------------------------
239. CM Select the indications for hospitalization of patients with chronic bronchitis:
a) [x] exacerbation episode with fever
b) [x] severe obstructive syndrome
c) [ ] leukocytosis 10.2 x 109 / l
d) [ ] ESR 16 mmHg
e) [x] installation of complications
---------------------------------------------------------------------
240. CM Non-pharmacological treatment of chronic bronchitis include:
a) [ ] Reduce eating spicy food
b) [ ] Reduction of alcohol drinking
c) [x] Avoiding professional factors
d) [x] Avoiding cold weather
e) [x] Smoking Cessation
-------------------------------------------------------------------
241. CM Indications for antimicrobial treatment in chronic bronchitis are:
a) [ ] mucous sputum
b) [x] yellow or green sputum
c) [x] increase of sputum amount
d) [x] fever
e) [ ] painful dry cough
--------------------------------------------------------------------
242. CM Choose bronchodilators used for treatment in chronic bronchitis:
a) [x] anticholinergic
b) [ ] chromones
c) [ ] leukotrienes
d) [x] β-adrenomimetics
e) [x] methylxanthines
---------------------------------------------------------------------
243. CM Select indications for oxygen therapy in patients with chronic bronchitis:
a) [x] permanent severe hypoxemia
b) [ ] anemia
c) [x] pulmonary hypertension
d) [x] secondary poliglobulia
e) [ ] the patient's unwillingness to stop smoking
--------------------------------------------------------------------
244. CM Which of the following pathological aspects are not characteristic for chronic obstructive bronchitis?
a) [x] non-caseating granuloma
b) [ ] goblet cell hyperplasia of small airways
c) [ ] smooth muscle hypertrophy in small airways
d) [ ] inflammatory cells in the small airway mucosa and submucosa
e) [x] exudative alveolitis
--------------------------------------------------------------------
245. CM Patient L., 50 y.o, complains of cough with purulent sputum, with the disease onset 10 years ago.
Physical examinations: lung auscultation - sibilant rales and rhonchi. Pulse 72 per minute, BP 120/70 mm
Hg. Chest X Ray: hyperinflation syndrome. He was diagnosed with chronic muco-purulent bronchitis,
infectious exacerbation. Possible complications are:
a) [x] pulmonary emphysema
b) [ ] acute mitral insufficiency
c) [x] bronchiectasis
d) [ ] pulmonary hemorrhage
e) [ ] myocardial infarction
---------------------------------------------------------------------
246. CS Lung damage in cystic fibrosis is caused by:
a) [ ] pulmonary edema
b) [x] bronchiectasis
c) [ ] major hemoptysis
d) [ ] pneumothorax
e) [ ] idiopathic pulmonary fibrosis
--------------------------------------------------------------------
247. CM Clinical manifestations of exocrine pancreatic insufficiency in patients with cystic fibrosis:
a) [x] steatorrhea
b) [ ] constipation
c) [x] weight loss
d) [ ] diabetes
e) [x] hypovitaminosis
--------------------------------------------------------------------
248. CM Select confirmatory tests of cystic fibrosis:
a) [ ] chest radiograph
b) [x] sweat test
c) [ ] CRP
d) [x] study the potential difference
e) [ ] pulmonary function tests
--------------------------------------------------------------------
249. CS Cigarette smoking is involved in the pathogenesis of chronic bronchitis by:
a) [ ] increasing cilia movement
b) [ ] mobilization of alveolar macrophages
c) [x] mucus gland hyperplasia
d) [ ] bronchial mucosa infiltration of eosinophils
e) [ ] release of cytokines
--------------------------------------------------------------------
250. CS Which of the following can be the cause of primary emphysema:
a) [x] alpha1-antitrypsin deficiency
b) [ ] alveolar tissue ischemia
c) [ ] mucopolysaccharides metabolism disorders
d) [ ] mitral insufficiency
e) [ ] pulmonary hypertension
---------------------------------------------------------------------
Allergology

251. CS What is the period of time after ingestion of an allergic food till the allergic reactions can occur?

a) [ ] within seconds

b) [x] from 30 minutes to 12 hours

c) [ ] from within minutes up until about two hours

d) [ ] within two days

e) [ ] within a week
---------------------------------------------------------------------
252. CS What are the foods that most commonly cause allergic reactions, EXCEPT?

a) [ ] peanuts

b) [ ] tree nuts

c) [ ] fish and shellfish

d) [x] sugar

e) [ ] dust
---------------------------------------------------------------------
253. CS What component of a food is responsible for an allergic reaction?

a) [ ] fat

b) [x] protein

c) [ ] carbohydrate

d) [ ] trans-fatty acids

e) [ ] glucose
---------------------------------------------------------------------

254. CS Angioedema is commonly associated with the use of:

a) [x] ACE inhibitors

b) [ ] Beta blockers

c) [ ] Loop diuretics

d) [ ] Alpha-receptor blockers

e) [ ] Calcium-channel blockers
---------------------------------------------------------------------
255. СS Which of the following statements about hereditary angioedema is true?

a) [ ] It is related to excessive amyloid deposition

b) [x] It is caused by a deficiency of the C1 esterase inhibitor


c) [ ] Attacks are triggered by antihistamines

d) [ ] Treatment involves dehydroepiandrosterone (DHEA) administration

e) [ ] It is has spontaneos resolving


---------------------------------------------------------------------
256. СS Which of the following medications is the most appropriate to use in the emergent treatment of
anaphylaxis?
a) [ ] Diphenhydramine

b) [ ] Isoproterenol

c) [x] Epinephrine

d) [ ] Prednisone

e) [ ] Atropine
---------------------------------------------------------------------
257. СS 28-year-old man presents to your clinic for evaluation of allergies. He has a long history consistent
with allergic rhinoconjunctivitis but also experiences urticarial lesions when he eats certain types of food.
He also occasionally has back pain from a recent sports injury. His medications include loratadine and low-
dose corticosteroids, which were prescribed by his primary care doctor, as well as ibuprofen and a daily
baby aspirin. You decide to perform skin testing on the patient. Which of the following interventions
should you recommend before performing epicutaneous testing?

a) [ ] The patient should discontinue aspirin 1 week before testing

b) [ ] The patient should discontinue loratadine 3 days before testing

c) [x] The patient should discontinue all the medication 1 week before testing

d) [ ] The patient should discontinue steroids 1 week before testing

e) [ ] The patient should discontinue ibuprofen 1 week before testing


---------------------------------------------------------------------
258. СS A 35-year-old man comes to your office with symptoms of nasal congestion and itchy eyes and throat.
He has been experiencing such symptoms for several years. Symptoms are present throughout the year, and
he is able to enjoy outdoor activities without worsening of the symptoms. He owns a cat, which does not
sleep in the same room with him. You order allergy skin testing and receive a report indicating a positive
response to dust mites and cat dander. Which of the following therapeutic interventions is the most
effective for this patient's symptoms?

a) [ ] Antihistamines

b) [x] Removal of the allergen from the patient's environment

c) [ ] Leukotriene receptor antagonists

d) [ ] Cromolyn sodium

e) [ ] Hyposensibilization therapy
---------------------------------------------------------------------
259. СS A 20-year-old woman comes to your office in early spring with complains of nasal congestion, runny
nose, and paroxysms of sneezing. She has been experiencing these symptoms for 10 days. She denies
having fever, cough, myalgias, or malaise. She states that she typically experiences similar symptoms in
September and October. Her medical history includes mild intermittent asthma since childhood. On
examination, she has dark rings under her eyes but no sinus tenderness. The nasal mucosa appears pale and
swollen, and there is clear rhinorrhea. Which of the following statements regarding this patient's
condition is false?

a) [ ] Nasal smear is likely to show a predominance of eosinophils

b) [ ] Her symptoms are the result of the IgE-mediated release of substances such as histamine that increase
epithelial permeability

c) [ ] Treatment of the condition can result in improvement of coexisting asthma in certain patients

d) [x] Although daily nasal steroid sprays can alleviate symptoms, they are generally not recommended because of
the risk of rhinitis medicamentosa

e) [ ] Immunotherapy can be employed in patients whose symptoms persist despite the avoidance of triggers and the
use of pharmacotherapy
---------------------------------------------------------------------

260. СS An 18-year-old man comes to clinic complaining of nasal stuffiness, left-sided maxillary tooth pain, and
postnasal drip. He has had these symptoms for more than 2 months. After the first 2 weeks of symptoms, he
was seen in a walk-in clinic and given a 5-day course of antibiotics, but his symptoms did not improve
significantly. He has not had fever or chills but complains that he wakes up with a sore throat on most days;
the throat pain tends to get better as the day goes on. On examination, he is afebrile, with mild tenderness to
palpation over the left maxilla and left forehead. His posterior oropharynx is slightly erythematous, with
yellowish drainage present, but there is no tonsillar exudate. Examination of the nares reveals hyperemic
mucosa and mucopurulent discharge. Which of the following statements regarding this patient's condition
is true?

a) [ ] Chronic sinusitis can be defined as sinus inflammation that persists for more than 3 weeks

b) [ ] Sinus radiographs are the procedure of choice for evaluating patients suspected of having chronic sinusitis

c) [ ] It is likely that anaerobic bacteria are the primary pathogens responsible for this patient's condition

d) [ ] Nasal culture has sufficient sensitivity and specificity to guide further antimicrobial therapy

e) [x] In patients with medically resistant chronic sinusitis, further workup for conditions such as cystic fibrosis,
structural abnormality, or fungal infection is appropriate
---------------------------------------------------------------------
261. СS A 43-year-old woman comes to your clinic complaining of nonhealing hives. She says that she started
having hives 6 weeks ago. The hives are mildly pruritic. When asked, she says that each individual hive
lasts for 2 or 3 days. Physical examination reveals multiple urticarial papules that do not blanch on
diascopy. You ask the patient to come back to your clinic after 3 days, and you confirm that some of the
lesions are still present. On the basis of this patient's history and physical examination, what would be
the next step in the workup?

a) [ ] Administer thyroid function tests

b) [ ] Perform an abdominal CT scan to rule out an intra-abdominal malignancy

c) [ ] Check sinus films, hepatitis serology, and stool studies for ova and parasites

d) [x] Perform a biopsy of one of the lesions

e) [ ] Check autoantibodies specific for systemic diseases


--------------------------------------------------------------------
262. CS A 34-year-old man presents to your clinic complaining of a recurrent, extremely pruritic rash on his
trunk and back. The rash started a few months ago. The rash comes and goes; the patient thinks it appears
when he exercises or eats spicy foods. Physical examination reveals multiple 2 to 3 mm scattered papular
wheals surrounded by large, erythematous flares. Which of the following is a likely diagnosis for this
patient?

a) [x] Cholinergic urticaria

b) [ ] Pressure urticaria

c) [ ] Idiopathic urticaria

d) [ ] Aquagenic urticaria

e) [ ] Food allergy
---------------------------------------------------------------------
263. СS While traveling in an airplane, a flight attendant asks you to evaluate a 44-year-old woman who has
sudden onset or urticaria, flushing, pruritus, shortness of breath, nausea, and vomiting. You learn that she
has a history of allergy to peanuts and that she may have eaten some without knowing it. On physical
examination, the patient is alert and is in moderate respiratory distress. Her blood pressure is 90/50 mm Hg,
and her heart rate 120 beats/min. She has diffuse inspiratory and expiratory wheezing, and she is
experiencing diffuse urticaria. What is the most appropriate treatment for this patient?

a) [ ] Administer oxygen and start I.V. steroids and I.V. fluids; the flight can be continued

b) [ ] Start an I.V., inject 1 mg of epinephrine I.V., and give I.V. steroids, I.V.fluids, and oxygen; the flight can be
continued

c) [x] Administer oxygen and epinephrine subcutaneously or intramuscularly,give I.V. antihistamines and I.V.
fluids, start steroids, and ask the pilot to land and transport the patient to an emergency care facility

d) [ ] Give oral antihistamines and oral prednisone and continue to watch the patient for further clinical deterioration

e) [ ] Give oral prednisone and continue to watch the patient for further clinical deterioration
---------------------------------------------------------------------
264. СS A 50-year-old woman is admitted to the hospital with a history of subjective fever of 2 weeks' duration.
The patient underwent mitral valve replacement surgery 5 years ago; in addition, she once experienced an
allergic reaction to penicillin, which she describes as a rash that occurred a few minutes after she received a
single dose of I.V. penicillin. Physical examination is remarkable for the presence of a diastolic and systolic
murmur in the mitral area. Transthoracic echocardiography shows a vegetation in the mitral valve. Blood
cultures show penicillin-sensitive viridans streptococci. On the basis of this patient's history of penicillin
allergy, which of the following would be the most appropriate course of action?

a) [ ] Start a cephalosporin

b) [x] Administer a penicillin skin test before starting antibiotics

c) [ ] Start a different β-lactam, such as imipenem

d) [ ] Start vancomycin

e) [ ] Start a different, non β-lactamic antibiotic


---------------------------------------------------------------------
265. СS A 45-year-old man with a history of diabetes and hypertension comes to the emergency department
with chest pain. He is found to have a myocardial infarction with ST segment depression. After 4 days in
the hospital, the patient has recurrent chest pain; ECG changes are consistent with further ischemia. His
cardiologist schedules cardiac catheterization; however, the patient says that 10 years ago, when he had an
abdominal CT scan, he had a bad reaction to intravenous contrast.
Which of the following would be the most appropriate approach in the management of this patient?

a) [x] Proceed with the catheterization; premedicate with corticosteroids and antihistamines; use nonionic contrast

b) [ ] Perform a contrast media radioallergosorbent test (RAST)

c) [ ] Continue with medical management

d) [ ] Obtain a contrast media skin test

e) [ ] Proceed another diagnostic test for ischemia


---------------------------------------------------------------------
266. СS A 36-year-old man is being evaluated for diarrhea. The patient has a 3-month history of diarrhea,
postprandial nausea and vomiting, and weight loss. There is no specific food that he can relate to his
symptoms. A complete blood count reveals anemia and eosinophilia. His serum IgE level is increased.
Small bowel biopsy reveals eosinophilic infiltration without vasculitis. Which of the following is the most
likely diagnosis for this patient?

a) [ ] Oral allergy syndrome

b) [ ] Churg-Strauss syndrome

c) [x] Eosinophilic gastroenteropathy

d) [ ] Immediate gastrointestinal hypersensitivity

e) [ ] Intestinal parasitic infection


---------------------------------------------------------------------
267. CS A 3-year-old boy is brought to your office by his mother, who relates that her son was diagnosed as
having peanut hypersensitivity 1 year ago. He developed urticaria and nasal congestion after ingestion of
peanuts. Since then, he has had two more episodes of hypersensitivity, with similar symptoms. His mother
asks about treatment. Which of the following is the most appropriate treatment for this patient?

a) [ ] Long-term use of antihistamines

b) [ ] Immunotherapy

c) [x] Elimination diet

d) [ ] Ketotifen

e) [ ] Long-term use of corticosteroids


---------------------------------------------------------------------
268. СS A 30-year-old woman presents with shortness of breath, angioedema, urticaria, and hypotension after
eating shellfish. She is successfully treated with epinephrine, intravenous fluids, and antihistamines. She
has a history of asthma and hypertension. She takes lisinopril and inhaled beclomethasone.
Radioallergosorbent testing reveals the presence of shellfish-specific IgE. Which of the following
statements regarding this patient's condition is the most accurate?

a) [ ] She had a type III hypersensitivity reaction

b) [ ] This allergy is likely to disappear in a few years


c) [ ] She should avoid other highly allergenic foods, such as peanuts and tree nuts, as well as shellfish

d) [x] She is at high risk for developing a more severe anaphylactic reaction in the future if she ingests shellfish

e) [ ] She is at high risk for developing bronchial asthma


---------------------------------------------------------------------
269. СS Which of the following drugs is a leukotriene antagonist/inhibitor?

a) [ ] Theophylline

b) [ ] Prednisone

c) [ ] Salmeterol

d) [ ] Terbutaline

e) [x] Zafirlukast
---------------------------------------------------------------------
270. СS A 48 year old woman is being evaluated for nonproductive cough that persist for 3 month. She
describes daily symptoms. She has no dyspnea, wheezing, fever, weight loss, no night sweats or recent
illness. She has recently traveled abroad. She wasn`t exposed no anyone who has been ill. She never
smoked. She was diagnosed with essential hypertension 6 months ago and take lisinopril daily. Physical
examination is unremarkable. No oral/pharyngeal exudate or drainage. Chest x ray is normal. Which is the
most appropriate management option for this patient this time?

a) [x] Discontinue lisinopril

b) [ ] Chest CT

c) [ ] Spirometry

d) [ ] Start antihistamine / decongestion combination

e) [ ] Start proton pump inhibitors


---------------------------------------------------------------------
271. CS Which of the following treatments is the treatment of choice for chronic allergic rhinitis?

a) [ ] Systemic antihistamines

b) [x] Intranasal steroids

c) [ ] Topical decongestants

d) [ ] Cromolyn sodium

e) [ ] Bee pollen extract


---------------------------------------------------------------------
272. CM What body systems can be affected by a food-allergy?

a) [x] gastrointestinal tract

b) [x] respiratory system

c) [ ] cardiovascular system

d) [x] skin
e) [ ] bones
---------------------------------------------------------------------
273. CM Symptoms of anaphylaxis can occur:

a) [x] shortly after coming in contact with an allergen

b) [ ] hours after coming in contact with an allergen

c) [ ] in special in the morning hours

d) [x] needs to be treated in ICU department

e) [ ] hours after coming in contact with an allergen


---------------------------------------------------------------------
274. CS Which of these are not likely to cause anaphylaxis:

a) [ ] medications

b) [x] pollen

c) [x] latex

d) [x] exercise

e) [ ] stinging insects
---------------------------------------------------------------------
275. CM If you are at risk for anaphylaxis, the best way to manage your condition is:

a) [x] avoid allergens that trigger symptoms

b) [x] carry autoinjectable epinephrine

c) [x] know how to use epinephrine

d) [ ] daily use of epinephrine

e) [ ] to use inhaled steroids


---------------------------------------------------------------------

--------------------------------------------------------------------
276. CM Which of these is a common allergy symptom?

a) [x] itchy nose, mouth, and eyes

b) [x] runny or stuffy nose

c) [ ] sneezing

d) [ ] itching skin rush


e) [ ] odynophagia
---------------------------------------------------------------------
277. CM Which of these tests can determine your allergy trigger?

a) [x] skin-prick test


b) [ ] CBC test

c) [ ] X-ray

d) [ ] total IgE
e) [x ] specific IgE
-----------------------------------------------------------------
278. CM Which of these can help treat seasonal allergies?

a) [x] antihistamines and decongestants

b) [x] corticosteroid nasal sprays

c) [x] allergy shots

d) [ ] antileucotriens
e) [ ] polipectomy
--------------------------------------------------------------------
279. CM What are the contraindications for immunotherapy?

a) [x] severe comorbidities

b) [ ] coexisting asthma and allergic rhinitis

c) [ ] allergies to many allergic triggers

d) [x] pregnancy

e) [ ] allergy to epinephrine
-------------------------------------------------------------------
280. CM What are the posible manifestations of drug allergy?

a) [x] respiratory syptoms

b) [x] gastrointestinal symptoms

c) [x] haemothological manifestations

d) [ ] Wegener granulematosis

e) [ ] Goodpasture syndrome
---------------------------------------------------------------------

You might also like